SAMPLE GENERAL THORACIC

A 55-year-old man presents with a 2-month history of a progressive unrelenting unproductive cough.  He can control the cough slightly with OTC cough suppressants.  Over the last 3 weeks he has also noticed mild dyspnea.  His wife also notes that his voice has become hoarse.  His weight is stable, and he denies any fever but does admit to some night sweats intermittently over the last few weeks. The patient smoked heavily (1.5 ppd for 20 years) but quit 10 years ago.  He has high blood pressure controlled with a single agent and high cholesterol controlled with diet alone.  He is very active working outdoors as a construction worker.  He lives in a suburb of Phoenix Arizona but travels to South America where he has relatives.   He has no history of any prior cancers or surgeries.   His PCP obtained a CXR that demonstrates a left hilar mass.  He was sent to you for further evaluation.

Question 1:

Please select the necessary next tests (up to 3) you would order/perform.  If you do not feel any testing is warranted, choose N/A.
  1. Complete pulmonary function testing
  2. Split Lung perfusion
  3. Oxygen consumption testing
  4. Echocardiogram
  5. Cardiac stress test
  6. Cardiology consultation
  7. Interventional pulmonary guided bx of the lung mass
  8. N/A

Question 2:

A bx of the mass reveals squamous cell carcinoma.  Imaging reveals a 4.3 cm central left upper lobe mass without any enlargement of mediastinal nodes.  There is no evidence of any metastasis on any imaging.  What intervention(s) will you recommend next?

 

SAMPLE CARDIAC

A 65 year-old man with hyperlipidemia, insulin-dependent diabetes, and obesity presents with intermittent chest tightness on exertion. His medication include aspirin, metoprolol, lisinopril, atorvastatin, metformin, and regular insulin. EKG shows normal sinus rhythm and non-specific findings. Troponin is 0.04 ng/ml.


Question 1:


What blood test(s) and/or non-invasive test(s) would you obtain at this point?  List up to 5.

 

Question 2:

After optimizing his medical status, the patient undergoes coronary angiography which demonstrates 80% stenosis of the proximal left anterior descending artery and no significant stenosis of the circumflex or right coronary arteries.  Over the next 3 weeks, the patient develops increasing chest pain on exertion.

What is/are the best approach(s) to treatment of this patient? List up to 2.